Jump to content

testing


imatfaal

can I add a poll to a long dead topic  

2 members have voted

  1. 1. has this worked

    • yes
      1
    • no
      1
  2. 2. questions two

    • this too
      1
    • nah
      1


Recommended Posts

[math] m\left(\sqrt{(x - j)^2 + (y - k)^2}\right) = n\left(\sqrt{(x - r)^2 + (y - s)^2}\right) [/math]

get rid of roots

[math] m^2\left((x - j)^2 + (y - k)^2\right) = n^2\left((x - r)^2 + (y - s)^2\right) [/math]

multiply out brackets

[math] m^2\left((x^2-2jx+j^2) + (y^2-2ky+k^2)\right) = n^2\left((x^2-2rx+r^2) + (y^2-2sy+s^2)\right) [/math]

multiply through

[math] m^2x^2-2m^2jx+m^2j^2 + m^2y^2-2m^2ky+m^2k^2 = n^2x^2-2n^2rx+n^2r^2 + n^2y^2-2n^2sy+n^2s^2 [/math]

gather like terms

[math] m^2x^2 - n^2x^2 + m^2y^2 - n^2y^2 -2m^2jx +2n^2rx -2m^2ky +2n^2sy = +n^2r^2 +n^2s^2 - m^2k^2 - m^2j^2[/math]

factorise a bit

[math] (m^2 - n^2)x^2 + (m^2 - n^2)y^2 -x(2m^2j -2n^2r) -y(2m^2k -2n^2s) = n^2r^2 +n^2s^2 - m^2k^2 - m^2j^2[/math]

this is an equation for a circle - the more general form is this

[math] (x-x_{centre})^2 +(y-y_{centre})^2 = radius^2[/math]

I am convinced I should be able to get one into the other - but at the moment I cannot.

This is why this is all in the sandbox - cos I could be up a gum tree

And thanks for the protip SchrHat - appreciated and implemented.

Edited by imatfaal
Link to comment
Share on other sites

How can I use this function?

 

If, on the odd occaision I have posted equations I do them using Outlook and copy and paste them

 

use the

[math] and [/math]

tags.

Most latex is accepted. There is a tutorial in the maths section.

You can also click on someone's equation and it will show you what is typed to make it (I have noticed it isn't always 100% accurate, but the mistakes I did notice were infrequent/some time ago).

The reply/multiquote buttons will also get you a copy of what someone else typed (eg. for correcting someone else's post), strip out the quote tags and surrounding text and modify the eqn to suit.

 

[math] (m^2 - n^2)x^2 + (m^2 - n^2)y^2 -x(2m^2j -2n^2r) -y(2m^2k -2n^2s) = n^2r^2 +n^2s^2 - m^2k^2 - m^2j^2[/math]

 

this is an equation for a circle - the more general form is this

 

[math] (x-x_{centre})^2 +(y-y_{centre})^2 = radius^2[/math]

 

I am convinced I should be able to get one into the other - but at the moment I cannot.

 

Almost there. Just complete the square(s).

It'll probably get even messier, either live with it for a couple more lines or play around with redefining some variables.

Link to comment
Share on other sites

 

Almost there. Just complete the square(s).

It'll probably get even messier, either live with it for a couple more lines or play around with redefining some variables.

 

I keep trying to complete the square and get too messed up - I know what it should come out as and just cannot get there

Link to comment
Share on other sites

Because I'm bored/curious/out of practise with algebra:

[math] (m^2 - n^2)x^2 + (m^2 - n^2)y^2 -x(2m^2j -2n^2r) -y(2m^2k -2n^2s) = n^2r^2 +n^2s^2 - m^2k^2 - m^2j^2[/math]

Tidy up a bit. Get some stuff we're not interested in on the rhs.

[math] (m -n)(m + n)x^2 + (m -n)(m + n)y^2 -x2j(m^2j - n^2r/j) -y2k(m^2 -n^2s/k) = n^2r^2 +n^2s^2 - m^2k^2 - m^2j^2[/math]

 

[math] (m - n)(m + n)x^2 + (m - n)(m + n)y^2 -x2j(m -n)(m + nr/j) -y2k(m - n)(m + ns/k) = n^2r^2 +n^2s^2 - m^2k^2 - m^2j^2[/math]

 

[math] (m + n)x^2 + (m + n)y^2 -x2(km + nr) -y2(km + ns) = \frac{n^2r^2 +n^2s^2 - m^2k^2 - m^2j^2}{m-n}[/math]

^Edit: Mistake. Have fun.

Now get the quadratics in a nice form:

[math] x^2 + y^2 -x\frac{2(km + nr)}{(m + n)} -y\frac{2(km + ns)}{(m + n)} = \frac{n^2r^2 +n^2s^2 - m^2k^2 - m^2j^2}{m^2-n^2}[/math]

For convenience, set:

[math]a = \frac{-(km + nr)}{(m + n)}[/math]

[math]b = \frac{-(km + ns)}{(m + n)}[/math]

 

Now:

[math] x^2 + y^2 +2ax + 2by = \frac{n^2r^2 +n^2s^2 - m^2k^2 - m^2j^2}{m^2-n^2}[/math]

 

Note that: [math] (x+a)^2 = x^2+2ax + a^2 \rightarrow x^2 + 2ax = (x+a)^2 - a^2[/math]

 

[math] \left(x - \frac{(km + nr)}{(m + n)}\right)^2 + \left(y - \frac{km + ns)}{(m + n)}\right)^2 = \frac{n^2r^2 +n^2s^2 - m^2k^2 - m^2j^2}{m^2-n^2} + \left(\frac{(km + nr)}{(m + n)}\right)^2 + \left(\frac{(km + nr)}{(m + n)}\right)^2 [/math]

 

And you're left with a horrific mess on the RHS which can be simplified, and a k I substituted instead of a j on line 4.

Edited by Schrödinger's hat
Link to comment
Share on other sites

Also, now that I actually turn my brain on, this seems a lot simpler.

[math] m\left(\sqrt{(x - j)^2 + (y - k)^2}\right) = n\left(\sqrt{(x - r)^2 + (y - s)^2}\right) [/math]

Set some variables:

[math] x' = x-j,\;y' = y-k,\; a = \left(\frac{m}{n}\right)^2\;b=j-r\;c = k-s[/math]

Now:

[math] a\left((x')^2 + (y')^2\right) = (x' + j - r)^2 + (y' + k - s)^2 [/math]

 

[math] a\left((x')^2 + (y')^2\right) = (x' + b)^2 + (y' + c)^2 [/math]

Set:

[math]d=a-1[/math]

And:

[math] d(x')^2 - 2bx' + d(y')^2 - 2cy' = b^2 + c^2 [/math]

[math] (x')^2 + \frac{-2b}{d}x' + (y')^2 + \frac{2c}{d}y' = \frac{b^2 + c^2}{d} [/math]

[math] (x' - \frac{b}{d})^2 + (y' - \frac{c}{d})^2 = \frac{b^2 + c^2}{d} + \left(\frac{b}{d}\right)^2 + \left(\frac{c}{d}\right)^2 [/math]

[math] (dx' - b)^2 + (dy' - c)^2 = db^2 + dc^2 + b^2 + c^2 = a(b^2 + c^2) [/math]

Finally:

[math] \left(x - j - \frac{b}{d}\right)^2 + \left(y - k - \frac{c}{d}\right)^2 = \frac{a}{d^2}(b^2 + c^2) [/math]

Resubbing in variables could be done before dividing by d to simplify things somewhat further.

And finally realise one is doing algebra for entertainment and go to find something productive to do...

Edited by Schrödinger's hat
Link to comment
Share on other sites

[math]x^2=y/3[/math]

 

 

Cool but how do I use a large operator?

 

fractions are \frac{}{} with the top in the first parentheses, and the bottom in the second.

Click on the images of the equations to see the source:

[math]
\oint_{s}\left(\iiint\limits_{\mbox{ham}}^{\mbox{fish}^a}  
\psi^{
\left[\begin{array}{ccc}
   	\frac{a}{b}    &\partial_s&\tfrac{\partial}{\partial  \pi}\\
  	1	&2    	&3\\
       4	&5    	&6
\end{array}\right]} \;  
d\mathcal{V}\right) d\mathbf{R}\rightarrow \pm \infty = 
\sqrt{3}[/math]  

Produces

[math] \oint_{s}\left(\iiint\limits_{\mbox{ham}}^{\mbox{fish}^a} \psi^{\left[\begin{array}{ccc}\frac{a}{b}&\partial_s&\tfrac{\partial}{\partial \pi}\\1&2&3\\4&5&6\end{array}\right]} \; d\mathcal{V}\right) d\mathbf{R}\rightarrow \pm \infty = \sqrt{3}[/math]

 

Most things in LaTeX are fairly straightforward. If I don't know how to do something, 4/5 times my first guess will be correct.

There's a tutorial in the maths section, as well as a bunch of examples in the sandbox and maths section (remember, click the images).

The forum often eats your formatting on the raw input. So if it's really complicated, paste it elsewhere (or get used to writing horrible unformatted messy latex :D ).

Link to comment
Share on other sites

Ah, in that case, you can totally get away with changing to a new coordinate system (centred on one of the circles and re-scaled by:

[math]\left(\tfrac{m}{n}\right)^2 - 1[/math]

so it's nice and pretty).

[math]x'=d(x-j),\;y'=d(y-k)[/math]

[math] \left(x' - b \right)^2 + \left(y' - c \right)^2 = a(b^2 + c^2) [/math]

 

Feel free to copy anything I did over at the appropriate time.

Edited by Schrödinger's hat
Link to comment
Share on other sites

Ah, in that case, you can totally get away with changing to a new coordinate system (centred on one of the circles and re-scaled by:

[math]\left(\tfrac{m}{n}\right)^2 - 1[/math]

so it's nice and pretty).

[math]x'=d(x-j),\;y'=d(y-k)[/math]

[math] \left(x' - b \right)^2 + \left(y' - c \right)^2 = a(b^2 + c^2) [/math]

 

Feel free to copy anything I did over at the appropriate time.

 

Yeah - I had almost done that to prove it to myself. Not with scaling but setting origin on one of the points and rotating till both point on an axis - got rid of a load of work.

Link to comment
Share on other sites

fractions are \frac{}{} with the top in the first parentheses, and the bottom in the second.

Click on the images of the equations to see the source:

[math] \oint_{s}\left(\iiint\limits_{\mbox{ham}}^{\mbox{fish}^a} \psi^{ \left[\begin{array}{ccc} \frac{a}{b} &\partial_s&\tfrac{\partial}{\partial \pi}\\ 1 &2 &3\\ 4 &5 &6 \end{array}\right]} \; d\mathcal{V}\right) d\mathbf{R}\rightarrow \pm \infty = \sqrt{3}[/math]

 

[math]6\frac{17}{19}= infinity * \sqrt{3.1}/{9}[/math]

 

[math]21^a}[/math]

Edited by Tres Juicy
Link to comment
Share on other sites

[math]\infty \mbox{infty} \div \mbox{div} \times \mbox{times} \star \mbox{star} \cdot \mbox{cdot}[/math]

[math]S p\,a\;c\quad e\qquad s[/math]

[math]21^a[/math]

 

It's very finnicky about parentheses, and rarely gives intelligible error messages when you misplace one (I don't think thhe forum gives intelligible error messages at all)

Edited by Schrödinger's hat
Link to comment
Share on other sites

Create an account or sign in to comment

You need to be a member in order to leave a comment

Create an account

Sign up for a new account in our community. It's easy!

Register a new account

Sign in

Already have an account? Sign in here.

Sign In Now
×
×
  • Create New...

Important Information

We have placed cookies on your device to help make this website better. You can adjust your cookie settings, otherwise we'll assume you're okay to continue.